Continuity and differentiability of a piecewise function

Click For Summary
The discussion focuses on the continuity and differentiability of the piecewise function defined as f(x) = x^2 for rational x and f(x) = x^4 for irrational x. It is established that the function is differentiable at x=0 and continuous only at x=±1, while being discontinuous elsewhere. To demonstrate discontinuity at points other than 0 and ±1, the approach involves using sequences of rational and irrational numbers converging to the same point. The necessity of explicitly constructing these sequences or justifying their existence based on prior knowledge is also debated. Overall, the key takeaway is the importance of limits in assessing continuity for this function.
lus1450
Messages
40
Reaction score
1

Homework Statement


Discuss the continuity and differentiability of
<br /> f(x) = <br /> \begin{cases}<br /> x^2 &amp; \text{if } x\in \mathbb{Q} \\<br /> x^4 &amp; \text{if } x\in \mathbb{R}\setminus \mathbb{Q}<br /> \end{cases}<br />

Homework Equations





The Attempt at a Solution


From the graph of ##f##, I can see that it will be differentiable at ##x=0##, and I think just continuous at ##x= \pm 1##. That is, discontinuous everywhere else. What would be the strategy in showing that points not equal to ##0\text{, } \pm 1## are discontinuous? I know I would take a sequence of rationals and a sequence of irrationals, but would I say consider ##x_n \in \mathbb{Q}## and ##y_n \in \mathbb{R} \setminus \mathbb{Q}## such that as ##n \rightarrow \infty##, ##x_n \rightarrow x## and ##y_n \rightarrow x##? I'm only guessing considering the function, since we'll have ##x^2 \ne x^4## for ##x \not\in \{0, \pm1\}##
 
Physics news on Phys.org
Zaculus said:

The Attempt at a Solution


From the graph of ##f##, I can see that it will be differentiable at ##x=0##, and I think just continuous at ##x= \pm 1##. That is, discontinuous everywhere else. What would be the strategy in showing that points not equal to ##0\text{, } \pm 1## are discontinuous? I know I would take a sequence of rationals and a sequence of irrationals, but would I say consider ##x_n \in \mathbb{Q}## and ##y_n \in \mathbb{R} \setminus \mathbb{Q}## such that as ##n \rightarrow \infty##, ##x_n \rightarrow x## and ##y_n \rightarrow x##? I'm only guessing considering the function, since we'll have ##x^2 \ne x^4## for ##x \not\in \{0, \pm1\}##
Yes, this is the right idea. In order for ##f## to be continuous at ##x##, one must have
$$\lim_{y \rightarrow x} f(y) = f(x)$$
In particular, the limit must exist. But if the limit exists, then we must get the same answer no matter how we approach ##x##. Therefore if ##x_n## and ##y_n## are two sequences converging to ##x##, and the limits ##\lim_{n\rightarrow \infty} f(x_n)## and ##\lim_{n\rightarrow \infty} f(y_n)## disagree, then ##\lim_{y \rightarrow x} f(y)## does not exist, so ##f## cannot be continuous at ##x##.
 
Thank you jbunniii. Now to make the proof more rigorous, would I need to find explicit ##x_n## and ##y_n## that converge to ##x##, or just "consider ##x_n## and ##y_n## which both go to ##x## as ##n \rightarrow \infty## ?
 
Zaculus said:
Thank you jbunniii. Now to make the proof more rigorous, would I need to find explicit ##x_n## and ##y_n## that converge to ##x##, or just "consider ##x_n## and ##y_n## which both go to ##x## as ##n \rightarrow \infty## ?
Good question. It depends on what knowledge is assumed in this problem set. If you can justify (based on what you have already covered in the course) writing something like "every ##x \in \mathbb{R}## is a limit point of both ##\mathbb{Q}## and ##\mathbb{R}\setminus\mathbb{Q}##, so there exist sequences ##x_n \in \mathbb{Q}## and ##y_n \in \mathbb{R}\setminus\mathbb{Q}## such that ##x_n \rightarrow x## and ##y_n \rightarrow x##" then that should suffice. Otherwise you might want to explicitly construct such sequences.
 
Question: A clock's minute hand has length 4 and its hour hand has length 3. What is the distance between the tips at the moment when it is increasing most rapidly?(Putnam Exam Question) Answer: Making assumption that both the hands moves at constant angular velocities, the answer is ## \sqrt{7} .## But don't you think this assumption is somewhat doubtful and wrong?

Similar threads

Replies
26
Views
2K
  • · Replies 6 ·
Replies
6
Views
2K
Replies
34
Views
3K
  • · Replies 51 ·
2
Replies
51
Views
4K
  • · Replies 5 ·
Replies
5
Views
2K
  • · Replies 13 ·
Replies
13
Views
2K
  • · Replies 11 ·
Replies
11
Views
2K
  • · Replies 4 ·
Replies
4
Views
2K
  • · Replies 20 ·
Replies
20
Views
3K
  • · Replies 8 ·
Replies
8
Views
3K